if you answer this i will give you brainlyist what is infinity x infinty bonus 2+2

Answers

Answer 1

Answer:

infinity^2

2+2=4

Step-by-step explanation:

Answer 2

Answer:

Infinity^2, 4

Step-by-step explanation:

Infinity x Infinity = Infinity^2

2 + 2 = 4

oo + oo = oooo


Related Questions

Can anyone help me with this

Answers

Answer:the 3rd one

Step-by-step explanation:

Find the slope ... ...

Answers

Answer:

-2

Step-by-step explanation:

rise over run = 4/-2 = -2

Which function has a greater rate of change?​

Answers

[tex]\huge\bold{\purple{\bold{⚡EuroNow⚡}}} [/tex]

[tex]\huge\underline\mathtt\colorbox{cyan}{Its easy}[/tex]

The function for EuroNow has a greater rate of change

15 points and brainliest if right!!

Answers

Answer:

y + 5x = -7

Step-by-step explanation:

Hello there?

y + 3 = -5(x + 2)

LHS

Open the brackets

= -5x - 10

RHS remains

Combine the two:

y + 3 = -5 - 10

Collecting the like terms and taking the value of x to the LHS

=> y + 5x = - 7

I hope this helps. Have a nice studies

How many 2 digit numbers have unit digit 6 but are not perfect squares

Answers

9514 1404 393

Answer:

  7

Step-by-step explanation:

Of the 9 2-digit numbers ending in 6, only 2 are perfect squares: 16 and 36. The other 7 are not perfect squares.

equation of the line that passes through the points (7, 6) and (-2, -3)?

Answers

Answer:

y=x-1

Step-by-step explanation:

Hi there!

We want to find the equation of the line that passes through the points (7, 6) and (-2, -3)

There are 3 ways to write the equation of the line:

Slope-intercept form, which is y=mx+b, where m is the slope and b is the y intercept Point-slope form, which is [tex]y-y_1=m(x-x_1)[/tex], where m is the slope and [tex](x_1, y_1)[/tex] is a pointStandard form, which is ax+by=c, where a, b, and c are free integer coefficients, but a and b cannot be 0, and a cannot be negative

The most common (and usually, the easiest way) would be slope-intercept form, so let's write it that way

First, we'll need to find the slope of the line

The slope can be found using the formula [tex]\frac{y_2-y_1}{x_2-x_1}[/tex], where [tex](x_1, y_1)[/tex] and [tex](x_2, y_2)[/tex] are points

We have two points, which is needed, but let's label their values in order to avoid any confusion:

[tex]x_1= 7\\y_1=6\\x_2=-2\\y_2=-3[/tex]

Now substitute these values into the formula to find the slope (m):

m=[tex]\frac{y_2-y_1}{x_2-x_1}[/tex]

m=[tex]\frac{-3-6}{-2-7}[/tex]

Subtract the numbers

m=[tex]\frac{-9}{-9}[/tex]

Divide

m=1

The slope of the line is 1

So far, we can write the equation of the line as this:

y=1x+b, or y=x+b

We'll need to find b

As the equation passes through both (7,6) and (-2, -3), we can use either one of them to solve for b

Taking (7, 6) for instance, substitute 7 as x and 6 as y:

6=1(7)+b

Multiply

6=7+b

Subtract 7 from both sides

-1=b

Now substitute -1 as b:

y=x-1

Hope this helps!

What is 32/48 in simplest form?

A.2/3

B.3/4

C.4/6

Answers

Answer: A) 2/3

--------------------------------

32/48

Divide the numerator and denominator by 16:

2/3

<3

Answer:

A. 2/3

Step-by-step explanation:

32 = 2 x 16

48 = 3 x 16

32 / 48

= ( 2 x 16 ) / ( 3 x 16 )

= 2 / 3

Given: m<4 + m<7 = 180°
Prove: c ll d
I need the statements and reasons

Answers

the horses name was friday

Answer:

Ummmm... we need a picture

explain for a brainlist !!!!!!!!!!

Answers

Step-by-step explanation:

cross multiply

so it will be

2y= -1x+5

2y+x=5

thats the only part i know

Help someone please just one and 2 that is all whoever answers 1 and 2 correctly, will get marked their answer the brainliest please​

Answers

Answer:

Step-by-step explanation:

1 7/8 divided by 2 2/5 if give good response = brainlist

Answers

Answer:

75/96 or in its simplest form, 25/32

Step-by-step explanation:

1. Convert both fractions into improper fractions

1 7/8 becomes 15/8 (1 × 8 = 8, 8 + 7 = 15)2 2/5 becomes 12/5 (2 × 5 = 10, 10 + 2 = 12)

2. Use KFC

Keep the first fraction the sameFlip the second fractionChange the sign from ÷ to ×

15/8 ÷ 12/5 = 15/8 × 5/12

15 × 5 = 758 × 12 = 9675/96 = 25/32 (divide the numerator and denomiator by 3)

Hope this help!

HELP PLEASE I CANT FIGURE THIS OUT

Answers

I don’t understand what’s the problem?

Answer:

the slope is 1/4. the equation would be y=1/4x-7 I think

Step-by-step explanation:

$4 is what percent of 50

Answers

Answer:

8%

Step-by-step explanation:

4/50 = 0.08 = 8%

Steps to solve "what percent is 4 of 50?" If you are using a calculator, simply enter 4÷50×100 which will give you 8 as the answer.

Select two national treasures saved by Dolley Madison.

Gilbert Stuart's portrait of George Washington
A bust of Thomas Jefferson
The Declaration of Independence
The U.S. Constitution

Answers

Answer:

Gilbert Stuart's portrait of George Washington The U.S. Constitution

Step-by-step explanation:

Answer:  Gilbert Stuart's portrait of George Washington

The Declaration of Independence

Step-by-step explanation:

When determining domain it is important to work from

Answers

Answer:

use graphs

Step-by-step explanation:

Another way to identify the domain and range of functions is by using graphs. Because the domain refers to the set of possible input values, the domain of a graph consists of all the input values shown on the x-axis. The range is the set of possible output values, which are shown on the y-axis.

what is the answer for the question A=B+C

Answers

Answer:

8=4+4

Step-by-step explanation:

Work out the nth term of the following sequence:
7, 16, 31, 52, 79

its a quadratic sequence and i worked out the second difference (6) so know its 3n² but i don't know the rest

Answers

Answer:

Step-by-step explanation:

Since it is given that it is quadratic it is of the form   f(n)=an^2 + bn+c.

Since we know a few terms we can plug in to get some equations:

f(1)= a +b+c = 7

f(2)=4a+2b+c = 16

f(3)=9a+3b+c = 31

Now you've got yourself a system of three equations which I trust you can solve.

I will say that I'm not sure that this is the most efficient solution(check out the link below which might have a better solve).

Here are some nice explanations with the "real" math notation for a slightly different but related problem:

https://math.stackexchange.com/questions/2345256/how-to-find-the-nth-term-of-quadratic-sequences

Good luck!

The perimeter of a rectangle measures 36 inches. If the width is 5 times the length, what is the width of the rectangle?

Answers

Answer:

the width of the rectangle is 15 inches ...

and the length is 3 inches

Answer:

180.

Step-by-step explanation:

to find this, you multiply 36 by 5, which gets you 180.

keyword: times

PLEASE ANSWER ASAP CORRECT ANSWER GETS BRAINLIST

Solve.

2 1/3−1/2x=−2/3

Enter your answer in the box.
x =

Answers

I guess the number 2 is to indicate the question...

Then the equation would be 1/3 - 1/2x = -2/3

1/3 - 1/2x - 1/3 = -2/3 - 1/3

-1/2x = -3/3

-1/2x = -1

-1/2x *2 = - 1*2

-x = -2

-x * -1 = -2 * -1

x = 2

Good luck

Perpendicular to y=-2x+1, but passes through (-4,4)

Answers

y=1/2x+6 is the new equation
Remember, the perpendicular slope is always the opposite reciprocal so the slope of this would be 1/2. Then, plug in the new slope along with the point in slope intercept form to solve for b.

What is the slope in the picture?

Answers

Answer:

positive; sence it's starting from the bottom and going up its positive

positive!! i think it is because its going up a hill and it started at the bottom!!! hope this helps

Which of the following describes the transformation from Figure 1 to Figure 2? On a coordinate plane, figure A B C D E has points (negative 3, 5), (negative 2, 5), (negative 1, 4), (negative 2, 3), (negative 5, 3). Figure A prime B prime C prime D prime E prime has points (2, 2), (3, 2), (4, 1), (3, 0), (0, 0). CLEAR CHECK translation 2 units to the right and 3 units down translation 3 units to the left and 2 units up translation 5 units to the right and 3 units down translation 5 units to the left and 3 units up

Answers

Answer:

a

Step-by-step explanation:

The transformation from Figure 1 to Figure 2 is:

The transformation of 5 units to the right and 3 units down.

Option C is the correct answer.

What is translation?

It is the movement of the shape in the left, right, up, and down directions.

The translated shape will have the same shape and shape.

There is a positive value when translated to the right and up.

There is a negative value when translated to the left and down.

We have,

A B C D E has points (-3, 5), (-2, 5), (-1, 4), (-2, 3), and (-5, 3).

A' B' C' D' E' has points (2, 2), (3, 2), (4, 1), (3, 0), and (0, 0).

Now,

A = (-3 + 5, 5 - 3) to A' = (2, 2)

B = (-2 + 5, 5 - 3) to B' = (3, 2)

C = (-1 + 5, 4 - 3) to C' = (4, 1)

D = (-2 + 5, 3 - 3) to A' = (3, 0)

E = (-5 + 5, 3 - 3) to E' = (0, 0)

We see that,

There is a translation of 5 units to the right and  3 units to the down.

Thus,

The transformation of 5 units to the right and 3 units down.

Learn more about translation here:

https://brainly.com/question/12463306

#SPJ1

A point $P$ is randomly selected from the square region with vertices at $(\pm 2, \pm 2)$. What is the probability that $P$ is within one unit of the origin

Answers

If P = (X, Y) is a point in the given square, then X and Y are i.i.d random variables each with distribution

[tex]\displaystyle P(X = x) = \begin{cases}\dfrac14 & \text{if } -2 \le x \le 2 \\ 0 & \text{otherwise}\end{cases}[/tex]

and so the joint density of X and Y is

[tex]\displaystyle P(X = x, Y = y) = \begin{cases}\dfrac1{16} & \text{if }-2 \le x \le 2 \text{ and } -2 \le y \le 2 \\ 0 &\text{otherwise}\end{cases}[/tex]

We want to find P(X² + Y² ≤ 1). Points that satisfy this inequality lie in the set

R = {(x, y) : -1 ≤ x ≤ 1 and -√(1 - x²) ≤ y ≤ √(1 - x²)}

but we can more easily describe the region in polar coordinates by setting

x = r cos(t) and y = r sin(t)

so that the set R is identical to

R' = {(r, t) : 0 ≤ r ≤ 1 and 0 ≤ t ≤ 2π}

Integrate the joint density over R' :

[tex]\displaystyle P(X^2 + Y^2 \le 1) = \iint_R \frac1{16} \, dx \, dy[/tex]

[tex]\displaystyle P(X^2 + Y^2 \le 1) = \iint_{R'} \frac r{16} \, dr \, dt[/tex]

[tex]\displaystyle P(X^2 + Y^2 \le 1) = \int_0^{2\pi} \int_0^1 \frac r{16} \, dr \, dt[/tex]

[tex]\displaystyle P(X^2 + Y^2 \le 1) = \int_0^{2\pi} \frac{1^2 - 0^2}{32} \, dt[/tex]

[tex]\displaystyle P(X^2 + Y^2 \le 1) = \frac1{32} \int_0^{2\pi} dt[/tex]

[tex]\displaystyle P(X^2 + Y^2 \le 1) = \frac{2\pi-0}{32}[/tex]

[tex]\displaystyle P(X^2 + Y^2 \le 1) = \boxed{\frac{\pi}{16}}[/tex]

If the length of each side of a cuboid decreases by 20%, find the percentage decrease in its volume.
Solution : (Identify the ratio of the length of one edge of the original cuboid and the length of one edge of the new cuboid)​

Answers

Answer:

Step-by-step explanation:

(1+25 /100) (1-20/100) (1-50/100)  <1

5/4 x 4/5 x 1/2 <1

Decrease in volume (in percent)

(1+25 /100) (1-20/100) (1-50/100)  x 100

=48.8%

maths lit November question paper paper 1 grade 11 2021?​

Answers

Answer:

can you be more clear i dont get the question (not trying to be mean)

Step-by-step explanation:

Ummmmmmmmmmm???????????

PROBLEM SOLVING A boat is traveling parallel to the shore along RT. When the

boat is at point R, the captain measures the angle to the lighthouse as 35º After the

boat has traveled 2.1 miles, the captain measures the angle to the lighthouse as 70°

Answers

The exterior angle theorem states that the sum of two opposite interior  angles is equal to the measure of the exterior angle.

The distance SL= 5.77 miles between the boat and the lighthouse after travelling 2.1 miles

According to the conditions given in the problem the exterior angle is 70 degrees and one of the opposite interior angle is 35 degrees.

m∠ L + m∠R= 70°

m∠ L +  35° = 70°

m∠ L = 70°-35°

m∠ L = 35°

If one interior angle is 35 degrees the other must also be 35 degrees to make a total of 70 degrees.

The sum of all angles of the triangle is always equal to 180 degrees.

So the third angle of the triangle will be

180°= 35°+35°+m ∠S

m∠S= 110°

From the triangle

Angle theta= m∠S= 110°

Fy= SL= height

Fx= RS = base

F= RL= hypotenuse

The line SL has to be found out.

Let the Fx= 2.1 miles

Then

Fx= Fcos ∅

But Cos   ∅ =  Cos 110°= -0.342

2.1=  F (-0.342)

F= - 6.140 = hypotenuse

Now the vertical component

Fy= Fsine theta

Fy= - 6.140 sine 110°

Fy= - 6.14×0.94

Fy= -5.77 miles

The negative sign indicates that it is in the opposite direction.

https://brainly.com/question/13729598

1. Mitch Hart's gross pay for this week is $425.78. He is single and claims 2 6 points

allowances. Using the percentage method of withholding, what amount

will Mitch's employer withhold from his pay for federal income tax?

Percentage Method of Withholding - Single Person

Each weekly allowance is $63.46.

Weekly Taxable The amount of income tax to

Wage

withhold is

Over But not

over

$51.00 $195.00 S0.00 plus (10% of the excess

over $51.00)

195.00 645.00 $14.40 plus (15% of the excess

over $195.00)

645.00 1482.00 $81.90 plus (25% of the excess

over $645.00)

Answers

Using the percentage method, the amount that Mitch's employer is going to withhold is $29.979

The total wage payment usually received by Mitch monthly is = $425.78Each weekly allowance is $63.46, It is noted that Mitch claims two allowances = $63.46 × 2= $126.92

Now, the amount subject to withholding from his total wage payment is;

= $(425.78 - 126.92)= $298.86

Provided that the weekly taxable wage is more than $195 but not up to $645; Then, by using the percentage method, the Federal income tax can be computed as:

= 15% of $(298.86 - 195) + $14.40= 15% of $103.86 + $14.40= $15.579 + $14.40= $29.979

Therefore, we can conclude that the amount that Mitch's employer is going to withhold is $29.979

Learn more about taxable income here:

https://brainly.com/question/17347618?referrer=searchResults

15. Describe and correct the error in finding the intercepts of the graph of the equation.
Х
6x + 9y
= 18
6x + 9(0) = 18
6x = 18
6x + Oy 18
6(0) + 9y = 18
9y =
= 18
X = 3
y = 2
The x-intercept is at (0,3), and the y-intercept is at (2, 0).

Answers

Answer:

Wrong order!

Step-by-step explanation:

There's a reason why coordinates are called an "ordered pair", and for every point (a, b) a is the x coordinate and b is the y coordinate. The calculations are correct, the way reporting them are not: the point should be (3.0) and (0,2) - in fact, all points on the x axis are of the form (p, 0) and all points on the y axis of the form (0,q)

if x and y vary inversely and x=2.5 when y =100 find x when y=25

Answers

Answer:

x is 10.

..............

What have you learned about geometric relationships?
PLEASE HELP 15 POINTS

Answers

Answer:

that you can define a tangent relationship between a line and an arc. If the adjoining elements change, the tangent relationship is maintained between the elements. Geometric relationships control how a sketch changes when edits are made.

Step-by-step explanation:

i took this i guess and it was gud

Other Questions
Joy has 500.00 to spend for food. She spent 98.75 on Monday, 73.75 on Tuesday and 50.25 on Wednesday. How much money was left? List at least eight different systems of the human body that you have learned about:1.2.3.4.5.6.7.8.Next, list your pet or favorite animal here:Does that animal have all eight systems? Record your answers below.Question # 1Long Text (essay)List at least eight different systems of the human body. Using your favorite animal, list which of the eight systems the animal does and does not have. how long to thaw a 15-pound turkey in the refrigerator whats the energy of motion A.thermal energy B. temperature C. heat D. kinetic energy E.potential energy To work out 20 + 5(3) you would multiply 5 and 3 first and then add the 20 to get a value of 35. Can you start at 35 and reverse those two steps to get back to 3? Explain the two steps you would take to do that. solve for x! please help giving BRAINLIEST. I got an answer, but I only got one mark help Sum of the Angles of a TriangleIf the three angles of a triangleare equal, what is themeasurement of each angle?BC[?] degrees Please help, thank you (NO LINKS!)Patsy has cheerleading practice every fourth day. She wants to be in the school play, but they have practice every sixth day. If both start on September 5th, what would be the next date she has to choose between cheerleading and play practice? Show your work and explain. the first shower head was used 20 gallons of water in 5 minutes. How many gallons of water will we use in one minute? PLEASE HELP!!!!! ILL MARK BRAINLIIST WHAT EVER IT IS CALLED PLEASE!!!!1. What theorem can be used to find a missing length in a right triangle?A. Base angle theoremB. Pythagorean theoremC. Midsegment theoremD. Triangle proportionality theorem2. Find the value of X in the figure below if LP is parallel to MOA. 10.7 unitsB. 3 unitsC. 5.5 unitsD. 6 units 613209x659832=?If you answer this correct congrats you're a genius (and yes its possible I solved myself) A movie theater sends out a coupon for 75% off the price of a ticket. What is a constitional monarchy? if there was 200 kids= 100% and 43 of them left what is the percent a respected mit professor had a simple 5-word rule for his classroom, and every company should follow it What are four solutions to the inequality r/3 -4? What are 5 examples of expansion and contraction? What are their benefits? jimmy rides home from karate practice with his friend and his friends mom each tuesday night. one night, his friends mom cant come, and he needs to get a ride from someone else. hes never driven the route before, but hes able to explain exactly where to turn to get home. this is an example of In a survey of 1400 people who owned a certain type of car, 700 said that would buy that type of car again. What percent do the people surveyed were satisfied with the car? Please help me with this and explain step by step on how I am able to get the answer so I could keep that in mind!